This site is not ready yet! The updated version will be available soon.
CS2103/T 2020 Jan-Apr
  • Full Timeline
  • Week 1 [Aug 12]
  • Week 2 [Aug 19]
  • Week 3 [Aug 26]
  • Week 4 [Sep 2]
  • Week 5 [Sep 9]
  • Week 6 [Sep 16]
  • Week 7 [Sep 30]
  • Week 8 [Oct 7]
  • Week 9 [Oct 14]
  • Week 10 [Oct 21]
  • Week 11 [Oct 28]
  • Week 12 [Nov 4]
  • Week 13 [Nov 11]
  • Textbook
  • Admin Info
  • Report Bugs
  • Forum
  • Instructors
  • Announcements
  • File Submissions
  • Tutorial Schedule
  • Java Coding Standard
  • Participation Marks List

  •  Individual Project (iP):
  • Individual Project Info
  • Duke Upstream Repo
  • iP Code Dashboard
  • iP Showcase

  •  Team Project (tP):
  • Team Project Info
  • Team IDs
  • Addressbook-level3
  • Addressbook-level 1,2,4
  • tP Code Dashboard
  • tP Showcase
  • Week 1 [Aug 12] - Topics

    Topics allocated to the week will appear in this tab.

    • [W1.1] OOP: Classes & Objects
    • [W1.1a] Paradigms → OOP → Introduction → What :

    • [W1.1b] Paradigms → OOP → Objects → What :

    • [W1.1c] Paradigms → OOP → Objects → Objects as Abstractions :

    • [W1.1d] Paradigms → OOP → Objects → Encapsulation of Objects :

    • [W1.1e] Paradigms → OOP → Classes → What :

    • [W1.1f] C++ to Java → Classes → Defining Classes :

    • [W1.1g] C++ to Java → Classes → Getters and setters :

    • [W1.1h] Paradigms → OOP → Classes → Class-Level Members :

    • [W1.1i] C++ to Java → Classes → Class-Level Members :

    • [W1.1j] Paradigms → OOP → Classes → Enumerations :

    • [W1.1k] C++ to Java → Miscellaneous Topics → Enumerations :

    • [W1.2] OOP Inheritance
    • [W1.2a] Paradigms → OOP → Inheritance → What :

    • [W1.2b] Paradigms → OOP → Inheritance → Overloading :

    • [W1.2c] Paradigms → OOP → Inheritance → Overriding :

    • [W1.2d] C++ to Java → Inheritance → Inheritance (Basics) :

    • [W1.3] OOP: Polymorphism

       Polymorphism

    • [W1.3a] Paradigms → OOP → Polymorphism → What :

    • [W1.3b] C++ to Java → Inheritance → Polymorphism :

       Abstract Classes

    • [W1.3c] Paradigms → OOP → Inheritance → Abstract Classes and Methods :

    • [W1.3d] C++ to Java → Inheritance → Abstract Classes and Methods :

       Interfaces

    • [W1.3e] Paradigms → OOP → Inheritance → Interfaces :

    • [W1.3f] C++ to Java → Inheritance → Interfaces :

       How Polymorphism Works

    • [W1.3g] Paradigms → OOP → Inheritance → Substitutability :

    • [W1.3h] Paradigms → OOP → Inheritance → Dynamic and Static Binding :

    • [W1.3i] Paradigms → OOP → Polymorphism → How :

    • [W1.4] Java: Collections
    • [W1.4a] C++ to Java → Collections → The Collections Framework :

    • [W1.4b] C++ to Java → Collections → The ArrayList Class :

    • [W1.4c] C++ to Java → Collections → The HashMap Class :

    • [W1.5] Exception Handling
    • [W1.5a] Implementation → Error Handling → Introduction → What :

    • [W1.5b] Implementation → Error Handling → Exceptions → What :

    • [W1.5c] C++ to Java → Exceptions → What are Exceptions? :

    • [W1.5d] Implementation → Error Handling → Exceptions → How :

    • [W1.5e] C++ to Java → Exceptions → How to Use Exceptions :

    • [W1.5f] Implementation → Error Handling → Exceptions → When :


    [W1.1] OOP: Classes & Objects

    W1.1a :

    Paradigms → OOP → Introduction → What

    Can describe OOP at a higher level

    Object-Oriented Programming (OOP) is a programming paradigm. A programming paradigm guides programmers to analyze programming problems, and structure programming solutions, in a specific way.

    Programming languages have traditionally divided the world into two parts—data and operations on data. Data is static and immutable, except as the operations may change it. The procedures and functions that operate on data have no lasting state of their own; they’re useful only in their ability to affect data.

    This division is, of course, grounded in the way computers work, so it’s not one that you can easily ignore or push aside. Like the equally pervasive distinctions between matter and energy and between nouns and verbs, it forms the background against which we work. At some point, all programmers—even object-oriented programmers—must lay out the data structures that their programs will use and define the functions that will act on the data.

    With a procedural programming language like C, that’s about all there is to it. The language may offer various kinds of support for organizing data and functions, but it won’t divide the world any differently. Functions and data structures are the basic elements of design.

    Object-oriented programming doesn’t so much dispute this view of the world as restructure it at a higher level. It groups operations and data into modular units called objects and lets you combine objects into structured networks to form a complete program. In an object-oriented programming language, objects and object interactions are the basic elements of design.

    -- Object-Oriented Programming with Objective-C, Apple

    Some other examples of programming paradigms are:

    Paradigm Programming Languages
    Procedural Programming paradigm C
    Functional Programming paradigm F#, Haskel, Scala
    Logic Programming paradigm Prolog

    Some programming languages support multiple paradigms.

    Java is primarily an OOP language but it supports limited forms of functional programming and it can be used to (although not recommended) write procedural code. e.g. se-edu/addressbook-level1

    JavaScript and Python support functional, procedural, and OOP programming.

    A) Choose the correct statements

    • a. OO is a programming paradigm
    • b. OO guides us in how to structure the solution
    • c. OO is mainly an abstraction mechanism
    • d. OO is a programming language
    • e. OO is modeled after how the objects in real world work

    B) Choose the correct statements

    • a. Java and C++ are OO languages
    • b. C language follows the Functional Programming paradigm
    • c. Java can be used to write procedural code
    • d. Prolog follows the Logic Programming paradigm

    A) (a)(b)(c)(e)

    Explanation: While many languages support the OO paradigm, OO is not a language itself.

    B) Choose the correct statement

    (a)(b)(c)(d)

    Explanation: C follows the procedural paradigm. Yes, we can write procedural code using OO languages e.g., AddressBook-level1.

    OO is a higher level mechanism than the procedural paradigm.

    True.

    Explanation: Procedural languages work at simple data structures (e.g., integers, arrays) and functions level. Because an object is an abstraction over data+related functions, OO works at a higher level.

    W1.1b :

    Paradigms → OOP → Objects → What

    Can describe how OOP relates to the real world

    Every object has both state (data) and behavior (operations on data). In that, they’re not much different from ordinary physical objects. It’s easy to see how a mechanical device, such as a pocket watch or a piano, embodies both state and behavior. But almost anything that’s designed to do a job does, too. Even simple things with no moving parts such as an ordinary bottle combine state (how full the bottle is, whether or not it’s open, how warm its contents are) with behavior (the ability to dispense its contents at various flow rates, to be opened or closed, to withstand high or low temperatures).

    It’s this resemblance to real things that gives objects much of their power and appeal. They can not only model components of real systems, but equally as well fulfill assigned roles as components in software systems.

    -- Object-Oriented Programming with Objective-C, Apple

    Object Oriented Programming (OOP) views the world as a network of interacting objects.

    A real world scenario viewed as a network of interacting objects:

    You are asked to find out the average age of a group of people Adam, Beth, Charlie, and Daisy. You take a piece of paper and pen, go to each person, ask for their age, and note it down. After collecting the age of all four, you enter it into a calculator to find the total. And then, use the same calculator to divide the total by four, to get the average age. This can be viewed as the objects You, Pen, Paper, Calculator, Adam, Beth, Charlie, and Daisy interacting to accomplish the end result of calculating the average age of the four persons. These objects can be considered as connected in a certain network of certain structure.

    OOP solutions try to create a similar object network inside the computer’s memory – a sort of a virtual simulation of the corresponding real world scenario – so that a similar result can be achieved programmatically.

    OOP does not demand that the virtual world object network follow the real world exactly.

    Our previous example can be tweaked a bit as follows:

    • Use an object called Main to represent your role in the scenario.
    • As there is no physical writing involved, we can replace the Pen and Paper with an object called AgeList that is able to keep a list of ages.

    Every object has both state (data) and behavior (operations on data).

    Object Real World? Virtual World? Example of State (i.e. Data) Examples of Behavior (i.e. Operations)
    Adam Name, Date of Birth Calculate age based on birthday
    Pen - Ink color, Amount of ink remaining Write
    AgeList - Recorded ages Give the number of entries, Accept an entry to record
    Calculator Numbers already entered Calculate the sum, divide
    You/Main Average age, Sum of ages Use other objects to calculate

    Every object has an interface and an implementation.

    Every real world object has:

    • an interface through which other objects can interact with it
    • an implementation that supports the interface but may not be accessible to the other object

    The interface and implementation of some real-world objects in our example:

    • Calculator: the buttons and the display are part of the interface; circuits are part of the implementation.
    • Adam: In the context of our 'calculate average age' example, the interface of Adam consists of requests that adam will respond to, e.g. "Give age to the nearest year, as at Jan 1st of this year" "State your name"; the implementation includes the mental calculation Adam uses to calculate the age which is not visible to other objects.

    Similarly, every object in the virtual world has an interface and an implementation.

    The interface and implementation of some virtual-world objects in our example:

    • Adam: the interface might have a method getAge(Date asAt); the implementation of that method is not visible to other objects.

    Objects interact by sending messages. Both real world and virtual world object interactions can be viewed as objects sending message to each other. The message can result in the sender object receiving a response and/or the receiver object’s state being changed. Furthermore, the result can vary based on which object received the message, even if the message is identical (see rows 1 and 2 in the example below).

    Examples:

    World Sender Receiver Message Response State Change
    Real You Adam "What is your name?" "Adam" -
    Real as above Beth as above "Beth" -
    Real You Pen Put nib on paper and apply pressure Makes a mark on your paper Ink level goes down
    Virtual Main Calculator (current total is 50) add(int i): int i = 23 73 total = total + 23

    Consider the following real-world scenario.

    Tom read a Software Engineering textbook (he has been assigned to read the book) and highlighted some of the text in it.

    Explain the following statements about OOP using the above scenario as an example.

    1. Object Oriented Programming (OOP) views the world as a network of interacting objects.
    2. Every object has both state (data) and behavior (operations on data).
    3. Every object has an interface and an implementation.
    4. Objects interact by sending messages.
    5. OOP does not demand that the virtual world object network follow the real world exactly.

    [1] Object Oriented Programming (OOP) views the world as a network of interacting objects.

    Interacting objects in the scenario: Tom, SE Textbook (Book for short), Text, (possibly) Highlighter

    objects usually match nouns in the description

    [2]Every object has both state (data) and behavior (operations on data).

    Object Examples of state Examples of behavior
    Tom memory of the text read read
    Book title show text
    Text font size get highlighted

    [3] Every object has an interface and an implementation.

    • Interface of an object consists of how other objects interact with it i.e., what other objects can do to that object
    • Implementation consist of internals of the object that facilitate the interactions but not visible to other objects.
    Object Examples of interface Examples of implementation
    Tom receive reading assignment understand/memorize the text read, remember the reading assignment
    Book show text, turn page how pages are bound to the spine
    Text read how characters/words are connected together or fixed to the book

    [4] Objects interact by sending messages.

    Examples:

    • Tom sends message turn page to the Book
    • Tom sends message show text to the Book. When the Book shows the Text, Tom sends the message read to the Text which returns the text content to Tom.
    • Tom sends message highlight to the Highlighter while specifying which Text to highlight. Then the Highlighter sends the message highlight to the specified Text.

    [5] OOP does not demand that the virtual world object network follow the real world exactly.

    Examples:

    • A virtual world simulation of the above scenario can omit the Highlighter object. Instead, we can teach Text to highlight themselves when requested.

    W1.1c :

    Paradigms → OOP → Objects → Objects as Abstractions

    Can explain the abstraction aspect of OOP

    The concept of Objects in OOP is an abstraction mechanism because it allows us to abstract away the lower level details and work with bigger granularity entities i.e. ignore details of data formats and the method implementation details and work at the level of objects.

    Abstraction is a technique for dealing with complexity. It works by establishing a level of complexity we are interested in, and suppressing the more complex details below that level.

    We can deal with a Person object that represents the person Adam and query the object for Adam's age instead of dealing with details such as Adam’s date of birth (DoB), in what format the DoB is stored, the algorithm used to calculate the age from the DoB, etc.

    W1.1d :

    Paradigms → OOP → Objects → Encapsulation of Objects

    Can explain the encapsulation aspect of OOP

    Encapsulation protects an implementation from unintended actions and from inadvertent access.
    -- Object-Oriented Programming with Objective-C, Apple

    An object is an encapsulation of some data and related behavior in terms of two aspects:

    1. The packaging aspect: An object packages data and related behavior together into one self-contained unit.

    2. The information hiding aspect: The data in an object is hidden from the outside world and are only accessible using the object's interface.

    Choose the correct statements

    • a. An object is an encapsulation because it packages data and behavior into one bundle.
    • b. An object is an encapsulation because it lets us think in terms of higher level concepts such as Students rather than student-related functions and data separately.

    Don't confuse encapsulation with abstraction.

    Choose the correct statement

    (a)

    Explanation: The second statement should be: An object is an abstraction encapsulation because it lets ...

    W1.1e :

    Paradigms → OOP → Classes → What

    Can explain the relationship between classes and objects

    Writing an OOP program is essentially writing instructions that the computer will uses to,

    1. create the virtual world of of the object network, and
    2. provide it the inputs to produce the outcome we want.

    A class contains instructions for creating a specific kind of objects. It turns out sometimes multiple objects keep the same type of data and have the same behavior because they are of the same kind. Instructions for creating a one kind (or ‘class’) of objects can be done once and that same instructions can be used to instantiate objects of that kind. We call such instructions a Class.

    Classes and objects in an example scenario

    Consider the example of writing an OOP program to calculate the average age of Adam, Beth, Charlie, and Daisy.

    Instructions for creating objects Adam, Beth, Charlie, and Daisy will be very similar because they are all of the same kind : they all represent ‘persons’ with the same interface, the same kind of data (i.e. name, DoB, etc.), and the same kind of behavior (i.e. getAge(Date), getName(), etc.). Therefore, we can have a class called Person containing instructions on how to create Person objects and use that class to instantiate objects Adam, Beth, Charlie, and Daisy.

    Similarly, we need classes AgeList, Calculator, and Main classes to instantiate one each of AgeList, Calculator, and Main objects.

    Class Objects
    Person objects representing Adam, Beth, Charlie, Daisy
    AgeList an object to represent the age list
    Calculator an object to do the calculations
    Main an object to represent you who manages the whole operation

    Consider the following scenario. If you were to simulate this in an OOP program, what are the classes and the objects you would use?

    A customer (name: John) gave a cheque to the Cashier (name: Peter) to pay for the LoTR and GoT books he bought.

    Class Objects
    Customer john
    Book LoTR, GoT
    Cheque checqueJohnGave
    Cashier peter

    Assume you are writing a CLI program called CityConnect for storing and querying distances between cities. The behavior is as follows:

    Welcome to CityConnect!
    
    Enter command: addroute Clementi BuonaVista 12
    Route from Clementi to BuonaVista with distance 12km added
    
    Enter command: getdistance Clementi BuonaVista
    Distance from Clementi to BuonaVista is 12
    
    Enter command: getdistance Clementi JurongWest
    No route exists from Clementi to JurongWest!
    
    Enter command: addroute Clementi JurongWest 24
    Route from Clementi to JurongWest with distance 24km added
    
    Enter command: getdistance Clementi JurongWest
    Distance from Clementi to JurongWest is 24
    
    Enter command: exit
    
    

    What classes would you have in your code if you write your program based on the OOP paradigm?

    One class you can have is Route

    W1.1f :

    C++ to Java → Classes → Defining Classes

    Can define Java classes

    As you know,

    • Defining a class introduces a new object type.
    • Every object belongs to some object type; that is, it is an instance of some class.
    • A class definition is like a template for objects: it specifies what attributes the objects have and what methods can operate on them.
    • The new operator instantiates objects, that is, it creates new instances of a class.
    • The methods that operate on an object type are defined in the class for that object.

    Here's a class called Time, intended to represent a moment in time. It has three attributes and no methods.

    public class Time {
        private int hour;
        private int minute;
        private int second;
    }
    

    You can give a class any name you like. The Java convention is to use PascalCase format for class names.

    The code is usually placed in a file whose name matches the class e.g., the Time class should be in a file named Time.java.

    When a class is public (e.g., the Time class in the above example) it can be used in other classes. But the instance variables that are private (e.g., the hour, minute and second attributes of the Time class) can only be accessed from inside the Time class.

    Constructors

    The syntax for constructors is similar to that of other methods, except:

    • The name of the constructor is the same as the name of the class.
    • The keyword static is omitted.
    • Do not return anything. A constructor returns the created object by default.

    When you invoke new, Java creates the object and calls your constructor to initialize the instance variables. When the constructor is done, it returns a reference to the new object.

    Here is an example constructor for the Time class:

    public Time() {
        hour = 0;
        minute = 0;
        second = 0;
    }
    

    This constructor does not take any arguments. Each line initializes an instance variable to zero (which in this example means midnight). Now you can create Time objects.

    Time time = new Time();

    Like other methods, constructors can be overloaded.

    You can add another constructor to the Time class to allow creating Time objects that are initialized to a specific time:

    public Time(int h, int m, int s) {
        hour = h;
        minute = m;
        second = s;
    }
    

    Here's how you can invoke the new constructor: Time justBeforeMidnight = new Time(11, 59, 59);

    this keyword

    The this keyword is a reference variable in Java that refers to the current object. You can use this the same way you use the name of any other object. For example, you can read and write the instance variables of this, and you can pass this as an argument to other methods. But you do not declare this, and you can’t make an assignment to it.

    In the following version of the constructor, the names and types of the parameters are the same as the instance variables (parameters don’t have to use the same names, but that’s a common style). As a result, the parameters shadow (or hide) the instance variables, so the keyword this is necessary to tell them apart.

    public Time(int hour, int minute, int second) {
        this.hour = hour;
        this.minute = minute;
        this.second = second;
    }
    

    this can be used to refer to a constructor of a class within the same class too.

    In this example the constructor Time() uses the this keyword to call its own overloaded constructor Time(int, int, int)

    public Time() {
        this(0, 0, 0); // call the overloaded constructor
    }
    
    public Time(int hour, int minute, int second) {
        // ...
    }
    
    
    Instance methods

    You can add methods to a class which can then be used from the objects of that class. These instance methods do not have the static keyword in the method signature. Instance methods can access attributes of the class.

    Here's how you can add a method to the Time class to get the number of seconds passed till midnight.

    public int secondsSinceMidnight() {
        return hour*60*60 + minute*60 + second;
    }
    

    Here's how you can use that method.

    Time t = new Time(0, 2, 5);
    System.out.println(t.secondsSinceMidnight() + " seconds since midnight!");
    

    Define a Circle class so that the code given below produces the given output. The nature of the class is a follows:

    • Attributes(all private):
      • int x, int y: represents the location of the circle
      • double radius: the radius of the circle
    • Constructors:
      • Circle(): initializes x, y, radius to 0
      • Circle(int x, int y, double radius): initializes the attributes to the given values
    • Methods:
      • getArea(): int
        Returns the area of the circle as an int value (not double). Calculated as 2 * PI * (radius)2
        You can convert to double to an int using (int) e.g., x = (int)2.25 gives x the value 2.
        You can use Math.PI to get the value of Pi
        You can use Math.pow() to raise a number to a specific power e.g., Math.pow(3, 2) calculates 3 2
    public class Main {
        public static void main(String[] args) {
            Circle c = new Circle();
    
            System.out.println(c.getArea());
            c = new Circle(1, 2, 5);
            System.out.println(c.getArea());
    
        }
    }
    

    0
    78
    
    • Put the Circle class in a file called Circle.java

    Partial solution:

    public class Circle {
        private int x;
        // ...
    
        public Circle(){
            this(0, 0, 0);
        }
    
        public Circle(int x, int y, double radius){
            this.x = x;
            // ...
        }
    
        public int getArea(){
            double area = Math.PI * Math.pow(radius, 2);
            return (int)area;
        }
    
    }
    

    W1.1g :

    C++ to Java → Classes → Getters and setters

    Can define getters and setters

    As the instance variables of Time are private, you can access them from within the Time class only. To compensate, you can provide methods to access attributes:

    public int getHour() {
        return hour;
    }
    
    public int getMinute() {
        return minute;
    }
    
    public int getSecond() {
        return second;
    }
    

    Methods like these are formally called “accessors”, but more commonly referred to as getters. By convention, the method that gets a variable named something is called getSomething.

    Similarly, you can provide setter methods to modify attributes of a Time object:

    public void setHour(int hour) {
        this.hour = hour;
    }
    
    public void setMinute(int minute) {
        this.minute = minute;
    }
    
    public void setSecond(int second) {
        this.second = second;
    }
    

    Consider the Circle class below:

    public class Circle {
        private int x;
        private int y;
        private double radius;
    
        public Circle(){
            this(0, 0, 0);
        }
    
        public Circle(int x, int y, double radius){
            this.x = x;
            this.y = y;
            this.radius = radius;
        }
    
        public int getArea(){
            double area = Math.PI * Math.pow(radius, 2);
            return (int)area;
        }
    
    }
    

    Update it as follows so that code given below produces the given output.

    • Add getter/setter methods for all three attributes
    • Update the setters and constructors such that if the radius supplied is negative, the code automatically set the radius to 0 instead.
    public class Main {
        public static void main(String[] args) {
            Circle c = new Circle(1,2, 5);
    
            c.setX(4);
            c.setY(5);
            c.setRadius(6);
            System.out.println("x      : " + c.getX());
            System.out.println("y      : " + c.getY());
            System.out.println("radius : " + c.getRadius());
            System.out.println("area   : " + c.getArea());
    
            c.setRadius(-5);
            System.out.println("radius : " + c.getRadius());
            c = new Circle(1, 1, -4);
            System.out.println("radius : " + c.getRadius());
    
        }
    }
    

    x      : 4
    y      : 5
    radius : 6.0
    area   : 113
    radius : 0.0
    radius : 0.0
    

    Partial solution:

    public Circle(int x, int y, double radius){
        setX(x);
        setY(y);
        setRadius(radius);
    }
    
    public void setRadius(double radius) {
        this.radius = Math.max(radius, 0);
    }
    

    W1.1h :

    Paradigms → OOP → Classes → Class-Level Members

    Can explain class-level members

    While all objects of a class has the same attributes, each object has its own copy of the attribute value.

    All Person objects have the Name attribute but the value of that attribute varies between Person objects.

    However, some attributes are not suitable to be maintained by individual objects. Instead, they should be maintained centrally, shared by all objects of the class. They are like ‘global variables’ but attached to a specific class. Such variables whose value is shared by all instances of a class are called class-level attributes.

    The attribute totalPersons should be maintained centrally and shared by all Person objects rather than copied at each Person object.

    Similarly, when a normal method is being called, a message is being sent to the receiving object and the result may depend on the receiving object.

    Sending the getName() message to Adam object results in the response "Adam" while sending the same message to the Beth object gets the response "Beth".

    However, there can be methods related to a specific class but not suitable for sending message to a specific object of that class. Such methods that are called using the class instead of a specific instance are called class-level methods.

    The method getTotalPersons() is not suitable to send to a specific Person object because a specific object of the Person class should not know about the total number of Person objects.

    Class-level attributes and methods are collectively called class-level members (also called static members sometimes because some programming languages use the keyword static to identify class-level members). They are to be accessed using the class name rather than an instance of the class.

    Which of these are suitable as class-level variables?

    • a. system: multi-player Pac Man game, Class: Player, variable: totalScore
    • b. system: eLearning system, class: Course, variable: totalStudents
    • c. system: ToDo manager, class: Task, variable: totalPendingTasks
    • d. system: any, class: ArrayList, variable: total (i.e., total items in a given ArrayList object)

    (c)

    Explanation: totalPendingTasks should not be managed by individual Task objects and therefore suitable to be maintained as a class-level variable. The other variables should be managed at instance level as their value varies from instance to instance. e.g., totalStudents for one Course object will differ from totalStudents of another.

    W1.1i :

    C++ to Java → Classes → Class-Level Members

    Can use class-level members

    The content below is an extract from -- Java Tutorial, with slight adaptations.

    When a number of objects are created from the same class blueprint, they each have their own distinct copies of instance variables. In the case of a Bicycle class, the instance variables are gear, and speed. Each Bicycle object has its own values for these variables, stored in different memory locations.

    Sometimes, you want to have variables that are common to all objects. This is accomplished with the static modifier. Fields that have the static modifier in their declaration are called static fields or class variables. They are associated with the class, rather than with any object. Every instance of the class shares a class variable, which is in one fixed location in memory. Any object can change the value of a class variable, but class variables can also be manipulated without creating an instance of the class.

    Suppose you want to create a number of Bicycle objects and assign each a serial number, beginning with 1 for the first object. This ID number is unique to each object and is therefore an instance variable. At the same time, you need a field to keep track of how many Bicycle objects have been created so that you know what ID to assign to the next one. Such a field is not related to any individual object, but to the class as a whole. For this you need a class variable, numberOfBicycles, as follows:

    public class Bicycle {
    
        private int gear;
        private int speed;
    
        // an instance variable for the object ID
        private int id;
    
        // a class variable for the number of Bicycle objects instantiated
        private static int numberOfBicycles = 0;
            ...
    }
    

    Class variables are referenced by the class name itself, as in Bicycle.numberOfBicycles This makes it clear that they are class variables.

    The Java programming language supports static methods as well as static variables. Static methods, which have the static modifier in their declarations, should be invoked with the class name, without the need for creating an instance of the class, as in ClassName.methodName(args)

    The static modifier, in combination with the final modifier, is also used to define constants. The final modifier indicates that the value of this field cannot change.For example, the following variable declaration defines a constant named PI, whose value is an approximation of pi (the ratio of the circumference of a circle to its diameter): static final double PI = 3.141592653589793;

    Here is an example with class-level variables and class-level methods:

    public class Bicycle {
    
        private int gear;
        private int speed;
    
        private int id;
    
        private static int numberOfBicycles = 0;
    
    
        public Bicycle(int startSpeed, int startGear) {
            gear = startGear;
            speed = startSpeed;
    
            numberOfBicycles++;
            id = numberOfBicycles;
        }
    
        public int getID() {
            return id;
        }
    
        public static int getNumberOfBicycles() {
            return numberOfBicycles;
        }
    
        public int getGear(){
            return gear;
        }
    
        public void setGear(int newValue) {
            gear = newValue;
        }
    
        public int getSpeed() {
            return speed;
        }
    
        // ...
    
    }
    

    Explanation of System.out.println(...):

    • out is a class-level public attribute of the System class.
    • println is a instance level method of the out object.

    Consider the Circle class below:

    public class Circle {
        private int x;
        private int y;
        private double radius;
    
        public Circle(){
            this(0, 0, 0);
        }
    
        public Circle(int x, int y, double radius){
            setX(x);
            setY(y);
            setRadius(radius);
        }
    
        public int getX() {
            return x;
        }
    
        public void setX(int x) {
            this.x = x;
        }
    
        public int getY() {
            return y;
        }
    
        public void setY(int y) {
            this.y = y;
        }
    
        public double getRadius() {
            return radius;
        }
    
        public void setRadius(double radius) {
            this.radius = Math.max(radius, 0);
        }
    
        public int getArea(){
            double area = Math.PI * Math.pow(radius, 2);
            return (int)area;
        }
    }
    

    Update it as follows so that code given below produces the given output.

    • Add a class-level getMaxRadius method that returns the maximum radius that has been used in all Circle objects created thus far.
    public class Main {
        public static void main(String[] args) {
            Circle c = new Circle();
            System.out.println("max radius used so far : " + Circle.getMaxRadius());
            c = new Circle(0, 0, 10);
            System.out.println("max radius used so far : " + Circle.getMaxRadius());
            c = new Circle(0, 0, -15);
            System.out.println("max radius used so far : " + Circle.getMaxRadius());
            c.setRadius(12);
            System.out.println("max radius used so far : " + Circle.getMaxRadius());
        }
    }
    

    max radius used so far : 0.0
    max radius used so far : 10.0
    max radius used so far : 10.0
    max radius used so far : 12.0
    

    You can use a static variable maxRadius to track the maximum value used for the radius attribute so far.

    Partial solution:

    public void setRadius(double radius) {
        this.radius = Math.max(radius, 0);
        if (maxRadius < this.radius){
            // ...
        }
    }
    

    W1.1j :

    Paradigms → OOP → Classes → Enumerations

    Can explain the meaning of enumerations

    An Enumeration is a fixed set of values that can be considered as a data type. An enumeration is often useful when using a regular data type such as int or String would allow invalid values to be assigned to a variable.

    Suppose you want a variable called priority to store the priority of something. There are only three priority levels: high, medium, and low. You can declare the variable priority as of type int and use only values 2, 1, and 0 to indication the three priority levels. However, this opens the possibility of an invalid values such as 9 being assigned to it. But if you define an enumeration type called Priority that has three values HIGH, MEDIUM, LOW only, a variable of type Priority will never be assigned an invalid value because the compiler is able to catch such an error.

    Priority: HIGH, MEDIUM, LOW

    W1.1k :

    C++ to Java → Miscellaneous Topics → Enumerations

    Can use Java enumerations

    You can define an enum type by using the enum keyword. Because they are constants, the names of an enum type's fields are in uppercase letters e.g., FLAG_SUCCESS by convention.

    Defining an enumeration to represent days of a week (code to be put in the Day.java file):

    public enum Day {
        SUNDAY, MONDAY, TUESDAY, WEDNESDAY,
        THURSDAY, FRIDAY, SATURDAY
    }
    

    Some examples of using the Day enumeration defined above:

    Day today = Day.MONDAY;
    Day[] holidays = new Day[]{Day.SATURDAY, Day.SUNDAY};
    
    switch (today) {
    case SATURDAY:
    case SUNDAY:
        System.out.println("It's the weekend");
        break;
    default:
        System.out.println("It's a week day");
    }
    

    Note that while enumerations are usually a simple set of fixed values, Java enumerations can have behaviors too, as explained in this tutorial from -- Java Tutorial

    Define an enumeration named Priority. Add the missing describe method to the code below so that it produces the output given.

    public class Main {
    
        // Add your method here
    
        public static void main(String[] args) {
            describe("Red", Priority.HIGH);
            describe("Orange", Priority.MEDIUM);
            describe("Blue", Priority.MEDIUM);
            describe("Green", Priority.LOW);
        }
    }
    

    Red indicates high priority
    Orange indicates medium priority
    Blue indicates medium priority
    Green indicates low priority
    

    Use a switch statement to select between possible values for Priority.

        public static void describe(String color, Priority p) {
            switch (p) {
                case LOW:
                    System.out.println(color + " indicates low priority");
                    break;
                // ...
            }
        }
    

    Code for the enumeration is given below:

    public enum Priority {
        HIGH, MEDIUM, LOW
    }
    

    [W1.2] OOP Inheritance

    W1.2a :

    Paradigms → OOP → Inheritance → What

    Can explain the meaning of inheritance

    The OOP concept Inheritance allows you to define a new class based on an existing class.

    For example, you can use inheritance to define an EvaluationReport class based on an existing Report class so that the EvaluationReport class does not have to duplicate data/behaviors that are already implemented in the Report class. The EvaluationReport can inherit the wordCount attribute and the print() method from the base class Report.

    • Other names for Base class: Parent class, Super class
    • Other names for Derived class: Child class, Sub class, Extended class

    A superclass is said to be more general than the subclass. Conversely, a subclass is said to be more specialized than the superclass.

    Applying inheritance on a group of similar classes can result in the common parts among classes being extracted into more general classes.

    Man and Woman behaves the same way for certain things. However, the two classes cannot be simply replaced with a more general class Person because of the need to distinguish between Man and Woman for certain other things. A solution is to add the Person class as a superclass (to contain the code common to men and woment) and let Man and Woman inherit from Person class.

    Inheritance implies the derived class can be considered as a sub-type of the base class (and the base class is a super-type of the derived class), resulting in an is a relationship.

    Inheritance does not necessarily mean a sub-type relationship exists. However, the two often go hand-in-hand. For simplicity, at this point let us assume inheritance implies a sub-type relationship.

    To continue the previous example,

    • Woman is a Person
    • Man is a Person

    Inheritance relationships through a chain of classes can result in inheritance hierarchies (aka inheritance trees).

    Two inheritance hierarchies/trees are given below. Note that the triangle points to the parent class. Observe how the Parrot is a Bird as well as it is an Animal.

    Multiple Inheritance is when a class inherits directly from multiple classes. Multiple inheritance among classes is allowed in some languages (e.g., Python, C++) but not in other languages (e.g., Java, C#).

    The Honey class inherits from the Food class and the Medicine class because honey can be consumed as a food as well as a medicine (in some oriental medicine practices). Similarly, a Car is an Vehicle, an Asset and a Liability.

    Which of these are correct?

    • a. Superclass is more general than the subclass.
    • b. Child class is more specialized than the parent class.
    • c. A class can inherit behavior from its ancestor classes (ancestor classes = classes above it in the inheritance hierarchy).
    • d. Code reuse can be one benefit of inheritance.
    • e. A change to the superclass will not affect its subclasses.

    (a) (b) (c) (d)

    Explanation: (e) is incorrect. Because subclasses inherit behavior from the superclass, any changes to the superclass could affect subclasses.

    W1.2b :

    Paradigms → OOP → Inheritance → Overloading

    Can explain method overloading

    Method overloading is when there are multiple methods with the same name but different type signatures. Overloading is used to indicate that multiple operations do similar things but take different parameters.

    Type Signature: The type signature of an operation is the type sequence of the parameters. The return type and parameter names are not part of the type signature. However, the parameter order is significant.

    Example:

    Method Type Signature
    int add(int X, int Y) (int, int)
    void add(int A, int B) (int, int)
    void m(int X, double Y) (int, double)
    void m(double X, int Y) (double, int)

    In the case below, the calculate method is overloaded because the two methods have the same name but different type signatures (String) and (int)

    • calculate(String): void
    • calculate(int): void

    W1.2c :

    Paradigms → OOP → Inheritance → Overriding

    Can explain method overriding

    Method overriding is when a sub-class changes the behavior inherited from the parent class by re-implementing the method. Overridden methods have the same name, same type signature, and same return type.

    Consider the following case of EvaluationReport class inheriting the Report class:

    Report methods EvaluationReport methods Overrides?
    print() print() Yes
    write(String) write(String) Yes
    read():String read(int):String No. Reason: the two methods have different signatures; This is a case of overloading (rather than overriding).

    Paradigms → OOP → Inheritance →

    Overloading

    Method overloading is when there are multiple methods with the same name but different type signatures. Overloading is used to indicate that multiple operations do similar things but take different parameters.

    Type Signature: The type signature of an operation is the type sequence of the parameters. The return type and parameter names are not part of the type signature. However, the parameter order is significant.

    Example:

    Method Type Signature
    int add(int X, int Y) (int, int)
    void add(int A, int B) (int, int)
    void m(int X, double Y) (int, double)
    void m(double X, int Y) (double, int)

    In the case below, the calculate method is overloaded because the two methods have the same name but different type signatures (String) and (int)

    • calculate(String): void
    • calculate(int): void

    Which of these methods override another method? A is the parent class. B inherits A.

    • a
    • b
    • c
    • d
    • e

    d

    Explanation: Method overriding requires a method in a child class to use the same method name and same parameter sequence used by one of its ancestors

    W1.2d :

    C++ to Java → Inheritance → Inheritance (Basics)

    Can use basic inheritance

    Given below is an extract from the -- Java Tutorial, with slight adaptations.

    A class that is derived from another class is called a subclass (also a derived class, extended class, or child class). The class from which the subclass is derived is called a superclass (also a base class or a parent class).

    A subclass inherits all the members (fields, methods, and nested classes) from its superclass. Constructors are not members, so they are not inherited by subclasses, but the constructor of the superclass can be invoked from the subclass.

    Every class has one and only one direct superclass (single inheritance), except the Object class, which has no superclass, . In the absence of any other explicit superclass, every class is implicitly a subclass of Object. Classes can be derived from classes that are derived from classes that are derived from classes, and so on, and ultimately derived from the topmost class, Object. Such a class is said to be descended from all the classes in the inheritance chain stretching back to Object. Java does not support multiple inheritance among classes.

    The java.lang.Object class defines and implements behavior common to all classes—including the ones that you write. In the Java platform, many classes derive directly from Object, other classes derive from some of those classes, and so on, forming a single hierarchy of classes.

    The keyword extends indicates one class inheriting from another.

    Here is the sample code for a possible implementation of a Bicycle class and a MountainBike class that is a subclass of the Bicycle:

    public class Bicycle {
    
        public int gear;
        public int speed;
    
        public Bicycle(int startSpeed, int startGear) {
            gear = startGear;
            speed = startSpeed;
        }
    
        public void setGear(int newValue) {
            gear = newValue;
        }
    
        public void applyBrake(int decrement) {
            speed -= decrement;
        }
    
        public void speedUp(int increment) {
            speed += increment;
        }
    
    }
    
    public class MountainBike extends Bicycle {
    
        // the MountainBike subclass adds one field
        public int seatHeight;
    
        // the MountainBike subclass has one constructor
        public MountainBike(int startHeight, int startSpeed, int startGear) {
            super(startSpeed, startGear);
            seatHeight = startHeight;
        }
    
        // the MountainBike subclass adds one method
        public void setHeight(int newValue) {
            seatHeight = newValue;
        }
    }
    

    A subclass inherits all the fields and methods of the superclass. In the example above, MountainBike inherits all the fields and methods of Bicycle and adds the field seatHeight and a method to set it.

    Accessing Superclass Members

    If your method overrides one of its superclass's methods, you can invoke the overridden method through the use of the keyword super. You can also use super to refer to a hidden field (although hiding fields is discouraged).

    Consider this class, Superclass and a subclass, called Subclass, that overrides printMethod():

    public class Superclass {
    
        public void printMethod() {
            System.out.println("Printed in Superclass.");
        }
    }
    
    public class Subclass extends Superclass {
    
        // overrides printMethod in Superclass
        public void printMethod() {
            super.printMethod();
            System.out.println("Printed in Subclass");
        }
        public static void main(String[] args) {
            Subclass s = new Subclass();
            s.printMethod();
        }
    }
    

    Printed in Superclass.
    Printed in Subclass
    

    Within Subclass, the simple name printMethod() refers to the one declared in Subclass, which overrides the one in Superclass. So, to refer to printMethod() inherited from Superclass, Subclass must use a qualified name, using super as shown.

    Subclass Constructors

    A subclass constructor can invoke the superclass constructor. Invocation of a superclass constructor must be the first line in the subclass constructor. The syntax for calling a superclass constructor is super() (which invokes the no-argument constructor of the superclass) or super(parameters) (to invoke the superclass constructor with a matching parameter list).

    The following example illustrates how to use the super keyword to invoke a superclass's constructor. Recall from the Bicycle example that MountainBike is a subclass of Bicycle. Here is the MountainBike (subclass) constructor that calls the superclass constructor and then adds some initialization code of its own (i.e., seatHeight = startHeight;):

    public MountainBike(int startHeight, int startSpeed, int startGear) {
        super(startSpeed, startGear);
        seatHeight = startHeight;
    }
    

    Note: If a constructor does not explicitly invoke a superclass constructor, the Java compiler automatically inserts a call to the no-argument constructor of the superclass. If the superclass does not have a no-argument constructor, you will get a compile-time error. Object does have such a constructor, so if Object is the only superclass, there is no problem.

    Access Modifiers (simplified)

    Access level modifiers determine whether other classes can use a particular field or invoke a particular method. Given below is a simplified version of Java access modifiers, assuming you have not yet started placing your classes in different packages i.e., all classes are placed in the root level. A full explanation of access modifiers is given in a later topic.

    There are two levels of access control:

    1. At the class level:

      • public: the class is visible to all other classes
      • no modifier: same as public

    2. At the member level:

      • public : the member is visible to all other classes
      • no modifier: same as public
      • protected: the member is visible to sub classes only
      • private: the member is not visible to other classes (but can be accessed in its own class)

    Background: Suppose we are creating a software to manage various tasks a person has to do. Two types of such tasks are,

    • Todos: i.e., things that needs to be done some day e.g., 'Read the book Lord of the Rings'
    • Deadlines: i.e., things to be done by a specific date/time e.g., 'Read the text book by Nov 25th'

    The Task class is given below:

    public class Task {
        protected String description;
    
        public Task(String description) {
            this.description = description;
        }
    
        public String getDescription() {
            return description;
        }
    }
    
    1. Write a Todo class that inherits from the Task class.
      • It should have an additional boolean field isDone to indicate whether the todo is done or not done.
      • It should have a isDone() method to access the isDone field and a setDone(boolean) method to set the isDone field.
    2. Write a Deadline class that inherits from the Todo class that you implemented in the previous step. It should have,
      • an additional String field by to store the details of when the task to be done e.g., Jan 25th 5pm
      • a getBy() method to access the value of the by field, and a corresponding setBy(String) method.
      • a constructor of the form Deadline(String description, String by)

    The expected behavior of the two classes is as follows:

    public class Main {
        public static void main(String[] args) {
            // create a todo task and print details
            Todo t = new Todo("Read a good book");
            System.out.println(t.getDescription());
            System.out.println(t.isDone());
    
            // change todo fields and print again
            t.setDone(true);
            System.out.println(t.isDone());
    
            // create a deadline task and print details
            Deadline d = new Deadline("Read textbook", "Nov 16");
            System.out.println(d.getDescription());
            System.out.println(d.isDone());
            System.out.println(d.getBy());
    
            // change deadline details and print again
            d.setDone(true);
            d.setBy("Postponed to Nov 18th");
            System.out.println(d.isDone());
            System.out.println(d.getBy());
        }
    }
    

    Read a good book
    false
    true
    Read textbook
    false
    Nov 16
    true
    Postponed to Nov 18th
    

    Todo class is given below. You can follow a similar approach for the Deadline class.

    public class Todo extends Task {
        protected boolean isDone;
    
        public Todo(String description) {
            super(description);
            isDone = false;
        }
    }
    

    [W1.3] OOP: Polymorphism


    Polymorphism

    W1.3a :

    Paradigms → OOP → Polymorphism → What

    Can explain OOP polymorphism

    Polymorphism:

    The ability of different objects to respond, each in its own way, to identical messages is called polymorphism. -- Object-Oriented Programming with Objective-C, Apple

    Polymorphism allows you to write code targeting superclass objects, use that code on subclass objects, and achieve possibly different results based on the actual class of the object.

    Assume classes Cat and Dog are both subclasses of the Animal class. You can write code targeting Animal objects and use that code on Cat and Dog objects, achieving possibly different results based on whether it is a Cat object or a Dog object. Some examples:

    • Declare an array of type Animal and still be able to store Dog and Cat objects in it.
    • Define a method that takes an Animal object as a parameter and yet be able to pass Dog and Cat objects to it.
    • Call a method on a Dog or a Cat object as if it is an Animal object (i.e., without knowing whether it is a Dog object or a Cat object) and get a different response from it based on its actual class e.g., call the Animal class' method speak() on object a and get a "Meow" as the return value if a is a Cat object and "Woof" if it is a Dog object.

    Polymorphism literally means "ability to take many forms".

    W1.3b :

    C++ to Java → Inheritance → Polymorphism

    Can use polymorphism in Java

    Java is a strongly-typed language which means the code works with only the object types that it targets.

    The following code PetShelter keeps a list of Cat objects and make them speak. The code will not work with any other type, for example, Dog objects.

    public class PetShelter {
        private static Cat[] cats = new Cat[]{
                new Cat("Mittens"),
                new Cat("Snowball")};
    
        public static void main(String[] args) {
            for (Cat c: cats){
                System.out.println(c.speak());
            }
        }
    }
    

    Mittens: Meow
    Snowball: Meow
    
    public class Cat {
        public Cat(String name) {
            super(name);
        }
    
        public String speak() {
            return name + ": Meow";
        }
    }
    

    This strong-typing can lead to unnecessary verbosity caused by repetitive similar code that do similar things with different object types.

    If the PetShelter is to keep both cats and dogs, you'll need two arrays and two loops:

    public class PetShelter {
        private static Cat[] cats = new Cat[]{
                new Cat("Mittens"),
                new Cat("Snowball")};
        private static Dog[] dogs = new Dog[]{
                new Dog("Spot")};
    
        public static void main(String[] args) {
            for (Cat c: cats){
                System.out.println(c.speak());
            }
            for(Dog d: dogs){
                System.out.println(d.speak());
            }
        }
    }
    

    Mittens: Meow
    Snowball: Meow
    Spot: Woof
    
    public class Dog {
        public Dog(String name) {
            super(name);
        }
    
        public String speak() {
            return name + ": Woof";
        }
    }
    

    A better way is to take advantage of polymorphism to write code that targets a superclass so that it works with any subclass objects.

    The PetShelter2 use one data structure to keep both types of animals and one loop to make them speak. The code targets the Animal superclass (assuming Cat and Dog inherits from the Animal class) instead of repeating the code for each animal type.

    public class PetShelter2 {
        private static Animal[] animals = new Animal[]{
                new Cat("Mittens"),
                new Cat("Snowball"),
                new Dog("Spot")};
    
        public static void main(String[] args) {
            for (Animal a: animals){
                System.out.println(a.speak());
            }
        }
    }
    

    Mittens: Meow
    Snowball: Meow
    Spot: Woof
    
    public class Animal {
    
        protected String name;
    
        public Animal(String name){
            this.name = name;
        }
        public String speak(){
            return name;
        }
    }
    
    public class Cat extends Animal {
        public Cat(String name) {
            super(name);
        }
    
        @Override
        public String speak() {
            return name + ": Meow";
        }
    }
    
    public class Dog extends Animal {
        public Dog(String name) {
            super(name);
        }
    
        @Override
        public String speak() {
            return name + ": Woof";
        }
    }
    

    Explanation: Because Java supports polymorphism, you can store both Cat and Dog objects in an array of Animal objects. Similarly, you can call the speak method on any Animal object (as done in the loop) and yet get different behavior from Cat objects and Dog objects.

    Suggestion: try to add an Animal object (e.g., new Animal("Unnamed")) to the animals array and see what happens.

    Polymorphic code is better in several ways:

    • It is shorter.
    • It is simpler.
    • It is more flexible (in the above example, the main method will work even if we add more animal types).

    The Main class below keeps a list of Circle and Rectangle objects and prints the area (as an int value) of each shape when requested.

    Add the missing variables/methods to the code below so that it produces the output given.

    public class Main {
        //TODO add your methods here
    
        public static void main(String[] args) {
            addShape(new Circle(5));
            addShape(new Rectangle(3, 4));
            addShape(new Circle(10));
            printAreas();
            addShape(new Rectangle(4, 4));
            printAreas();
        }
    }
    

    78
    12
    314
    78
    12
    314
    16
    

    Circle class and Rectangle class is given below but you'll need to add a parent class Shape:

    public class Circle {
    
        private int radius;
    
        public Circle(int radius) {
            this.radius = radius;
        }
    
        public int area() {
            return (int)(Math.PI * radius * radius);
        }
    }
    
    public class Rectangle {
        private int height;
        private int width;
    
        public Rectangle(int height, int width){
            this.height = height;
            this.width = width;
        }
    
        public int area() {
            return height * width;
        }
    }
    

    You may use an array of size 100 to store the shapes.

    public class Main {
        private static Shape[] shapes = new Shape[100];
        private static int shapeCount = 0;
    
        public static void addShape(Shape s){
            shapes[shapeCount] = s;
            shapeCount++;
        }
    
        // ...
    
    }
    

    Abstract Classes

    W1.3c :

    Paradigms → OOP → Inheritance → Abstract Classes and Methods

    Can implement abstract classes

    Abstract Class: A class declared as an abstract class cannot be instantiated, but it can be subclassed.

    You can declare a class as abstract when a class is merely a representation of commonalities among its subclasses in which case it does not make sense to instantiate objects of that class.

    The Animal class that exist as a generalization of its subclasses Cat, Dog, Horse, Tiger etc. can be declared as abstract because it does not make sense to instantiate an Animal object.

    Abstract Method: An abstract method is a method signature without a method implementation.

    The move method of the Animal class is likely to be an abstract method as it is not possible to implement a move method at the Animal class level to fit all subclasses because each animal type can move in a different way.

    A class that has an abstract method becomes an abstract class because the class definition is incomplete (due to the missing method body) and it is not possible to create objects using an incomplete class definition.

    W1.3d :

    C++ to Java → Inheritance → Abstract Classes and Methods

    Can use abstract classes and methods

    In Java, an abstract method is declared with the keyword abstract and given without an implementation. If a class includes abstract methods, then the class itself must be declared abstract.

    The speak method in this Animal class is abstract. Note how the method signature ends with a semicolon and there is no method body. This makes sense as the implementation of the speak method depends on the type of the animal and it is meaningless to provide a common implementation for all animal types.

    public abstract class Animal {
    
        protected String name;
    
        public Animal(String name){
            this.name = name;
        }
        public abstract String speak();
    }
    

    As one method of the class is abstract, the class itself is abstract.

    An abstract class is declared with the keyword abstract. Abstract classes can be used as reference type but cannot be instantiated.

    This Account class has been declared as abstract although it does not have any abstract methods. Attempting to instantiate Account objects will result in a compile error.

    public abstract class Account {
    
        int number;
    
        void close(){
            //...
        }
    }
    

    Account a; OK to use as a type
    a = new Account(); Compile error!

    In Java, even a class that does not have any abstract methods can be declared as an abstract class.

    When an abstract class is subclassed, the subclass should provides implementations for all of the abstract methods in its superclass or else the subclass must also be declared abstract.

    The Feline class below inherits from the abstract class Animal but it does not provide an implementation for the abstract method speak. As a result, the Feline class needs to be abstract too.

    public abstract class Feline extends Animal {
        public Feline(String name) {
            super(name);
        }
    
    }
    

    The DomesticCat class inherits the abstract Feline class and provides the implementation for the abstract method speak. As a result, it need not be (but can be) declared as abstract.

    public class DomesticCat extends Feline {
        public DomesticCat(String name) {
            super(name);
        }
    
        @Override
        public String speak() {
            return "Meow";
        }
    }
    
    • Animal a = new Feline("Mittens");
      Compile error! Feline is abstract.
    • Animal a = new DomesticCat("Mittens");
      OK. DomesticCat can be instantiated and assigned to a variable of Animal type (the assignment is allowed by polymorphism).

    The Main class below keeps a list of Circle and Rectangle objects and prints the area (as an int value) of each shape when requested.

    public class Main {
        private static Shape[] shapes = new Shape[100];
        private static int shapeCount = 0;
    
        public static void addShape(Shape s){
            shapes[shapeCount] = s;
            shapeCount++;
        }
    
        public static void printAreas(){
            for (int i = 0; i < shapeCount; i++){
                shapes[i].print();
            }
        }
    
        public static void main(String[] args) {
            addShape(new Circle(5));
            addShape(new Rectangle(3, 4));
            addShape(new Circle(10));
            addShape(new Rectangle(4, 4));
            printAreas();
        }
    }
    

    Circle of area 78
    Rectangle of area 12
    Circle of area 314
    Rectangle of area 16
    

    Circle class and Rectangle class is given below:

    public class Circle extends Shape {
    
        private int radius;
    
        public Circle(int radius) {
            this.radius = radius;
        }
    
        @Override
        public int area() {
            return (int)(Math.PI * radius * radius);
        }
    
        @Override
        public void print() {
            System.out.println("Circle of area " + area());
        }
    }
    
    public class Rectangle extends Shape {
        private int height;
        private int width;
    
        public Rectangle(int height, int width){
            this.height = height;
            this.width = width;
        }
    
        @Override
        public int area() {
            return height * width;
        }
    
        @Override
        public void print() {
            System.out.println("Rectangle of area " + area());
        }
    }
    

    Add the missing Shape class as an abstract class with two abstract methods.

    public abstract class Shape {
    
        public abstract int area();
        // ...
    }
    

    Choose the correct statements about Java abstract classes and concrete classes.

    • a. A concrete class can contain an abstract method.
    • b. An abstract class can contain concrete methods.
    • c. An abstract class need not contain any concrete methods.
    • d. An abstract class cannot be instantiated.

    (b)(c)(d)

    Explanation: A concrete class cannot contain even a single abstract method.


    Interfaces

    W1.3e :

    Paradigms → OOP → Inheritance → Interfaces

    Can explain interfaces

    An interface is a behavior specification i.e. a collection of method specifications. If a class implements the interface, it means the class is able to support the behaviors specified by the said interface.

    There are a number of situations in software engineering when it is important for disparate groups of programmers to agree to a "contract" that spells out how their software interacts. Each group should be able to write their code without any knowledge of how the other group's code is written. Generally speaking, interfaces are such contracts. --Oracle Docs on Java

    Suppose SalariedStaff is an interface that contains two methods setSalary(int) and getSalary(). AcademicStaff can declare itself as implementing the SalariedStaff interface, which means the AcademicStaff class must implement all the methods specified by the SalariedStaff interface i.e., setSalary(int) and getSalary().

    A class implementing an interface results in an is-a relationship, just like in class inheritance.

    In the example above, AcademicStaff is a SalariedStaff. An AcademicStaff object can be used anywhere a SalariedStaff object is expected e.g. SalariedStaff ss = new AcademicStaff().

    W1.3f :

    C++ to Java → Inheritance → Interfaces

    Can use interfaces in Java

    The text given in this section borrows some explanations and code examples from the -- Java Tutorial.

    In Java, an interface is a reference type, similar to a class, mainly containing method signatures. Defining an interface is similar to creating a new class except it uses the keyword interface in place of class.

    Here is an interface named DrivableVehicle that defines methods needed to drive a vehicle.

    public interface DrivableVehicle {
        void turn(Direction direction);
        void changeLanes(Direction direction);
        void signalTurn(Direction direction, boolean signalOn);
        // more method signatures
    }
    

    Note that the method signatures have no braces ({ }) and are terminated with a semicolon.

    Interfaces cannot be instantiated—they can only be implemented by classes. When an instantiable class implements an interface, indicated by the keyword implements, it provides a method body for each of the methods declared in the interface.

    Here is how a class CarModelX can implement the DrivableVehicle interface.

    public class CarModelX implements DrivableVehicle {
    
        @Override
        public void turn(Direction direction) {
           // implementation
        }
    
        // implementation of other methods
    }
    

    An interface can be used as a type e.g., DrivableVechile dv = new CarModelX();.

    Interfaces can inherit from other interfaces using the extends keyword, similar to a class inheriting another.

    Here is an interface named SelfDrivableVehicle that inherits the DrivableVehicle interface.

    public interface SelfDrivableVehicle extends DrivableVehicle {
       void goToAutoPilotMode();
    }
    

    Note that the method signatures have no braces and are terminated with a semicolon.

    Furthermore, Java allows multiple inheritance among interfaces. A Java interface can inherit multiple other interfaces. A Java class can implement multiple interfaces (and inherit from one class).

    The design below is allowed by Java. In case you are not familiar with UML notation used: solid lines indicate normal inheritance; dashed lines indicate interface inheritance; the triangle points to the parent.

    1. Staff interface inherits (note the solid lines) the interfaces TaxPayer and Citizen.
    2. TA class implements both Student interface and the Staff interface.
    3. Because of point 1 above, TA class has to implement all methods in the interfaces TaxPayer and Citizen.
    4. Because of points 1,2,3, a TA is a Staff, is a TaxPayer and is a Citizen.

    Interfaces can also contain constants and static methods.

    C++ to Java → Miscellaneous Topics →

    Constants

    Java does not directly support constants. The convention is to use a static final variable where a constant is needed. The static modifier causes the variable to be available without instantiating an object. The final modifier causes the variable to be unchangeable. Java constants are normally declared in ALL CAPS separated by underscores.

    Here is an example of a constant named MAX_BALANCE which can be accessed as Account.MAX_BALANCE.

    public class Account{
    
      public static final double MAX_BALANCE = 1000000.0;
    
    }
    

    Math.PI is an example constant that comes with Java.

    This example adds a constant MAX_SPEED and a static method isSpeedAllowed to the interface DrivableVehicle.

    public interface DrivableVehicle {
    
        int MAX_SPEED = 150;
    
        static boolean isSpeedAllowed(int speed){
            return speed <= MAX_SPEED;
        }
    
        void turn(Direction direction);
        void changeLanes(Direction direction);
        void signalTurn(Direction direction, boolean signalOn);
        // more method signatures
    }
    

    Interfaces can contain default method implementations and nested types. They are not covered here.

    The Main class below passes a list of Printable objects (i.e., objects that implement the Printable interface) for another method to be printed.

    public class Main {
    
        public static void printObjects(Printable[] items) {
            for (Printable p : items) {
                p.print();
            }
        }
    
        public static void main(String[] args) {
            Printable[] printableItems = new Printable[]{
                    new Circle(5),
                    new Rectangle(3, 4),
                    new Person("James Cook")};
    
            printObjects(printableItems);
        }
    }
    

    Circle of area 78
    Rectangle of area 12
    Person of name James Cook
    

    Classes Shape, Circle, and Rectangle are given below:

    public abstract class Shape {
    
        public abstract int area();
    }
    
    public class Circle extends Shape implements Printable {
    
        private int radius;
    
        public Circle(int radius) {
            this.radius = radius;
        }
    
        @Override
        public int area() {
            return (int)(Math.PI * radius * radius);
        }
    
        @Override
        public void print() {
            System.out.println("Circle of area " + area());
        }
    }
    
    public class Rectangle extends Shape implements Printable {
        private int height;
        private int width;
    
        public Rectangle(int height, int width){
            this.height = height;
            this.width = width;
        }
    
        @Override
        public int area() {
            return height * width;
        }
    
        @Override
        public void print() {
            System.out.println("Rectangle of area " + area());
        }
    }
    

    Add the missing Printable interface. Add the missing methods of the Person class given below.

    public class Person implements Printable {
    
        private String name;
    
        // todo: add missing methods
    }
    
    public interface Printable {
        //...
    }
    

    How Polymorphism Works

    W1.3g :

    Paradigms → OOP → Inheritance → Substitutability

    Can explain substitutability

    Every instance of a subclass is an instance of the superclass, but not vice-versa. As a result, inheritance allows substitutability : the ability to substitute a child class object where a parent class object is expected.

    an Academic is an instance of a Staff, but a Staff is not necessarily an instance of an Academic. i.e. wherever an object of the superclass is expected, it can be substituted by an object of any of its subclasses.

    The following code is valid because an AcademicStaff object is substitutable as a Staff object.

    Staff staff = new AcademicStaff (); // OK
    

    But the following code is not valid because staff is declared as a Staff type and therefore its value may or may not be of type AcademicStaff, which is the type expected by variable academicStaff.

    Staff staff;
    ...
    AcademicStaff academicStaff = staff; // Not OK
    

    W1.3h :

    Paradigms → OOP → Inheritance → Dynamic and Static Binding

    Can explain dynamic and static binding

    Dynamic Binding ( aka late binding) : a mechanism where method calls in code are resolved at runtime, rather than at compile time.

    Overridden methods are resolved using dynamic binding, and therefore resolves to the implementation in the actual type of the object.

    Paradigms → OOP → Inheritance →

    Overriding

    Method overriding is when a sub-class changes the behavior inherited from the parent class by re-implementing the method. Overridden methods have the same name, same type signature, and same return type.

    Consider the following case of EvaluationReport class inheriting the Report class:

    Report methods EvaluationReport methods Overrides?
    print() print() Yes
    write(String) write(String) Yes
    read():String read(int):String No. Reason: the two methods have different signatures; This is a case of overloading (rather than overriding).

    Paradigms → OOP → Inheritance →

    Overloading

    Method overloading is when there are multiple methods with the same name but different type signatures. Overloading is used to indicate that multiple operations do similar things but take different parameters.

    Type Signature: The type signature of an operation is the type sequence of the parameters. The return type and parameter names are not part of the type signature. However, the parameter order is significant.

    Example:

    Method Type Signature
    int add(int X, int Y) (int, int)
    void add(int A, int B) (int, int)
    void m(int X, double Y) (int, double)
    void m(double X, int Y) (double, int)

    In the case below, the calculate method is overloaded because the two methods have the same name but different type signatures (String) and (int)

    • calculate(String): void
    • calculate(int): void

    Which of these methods override another method? A is the parent class. B inherits A.

    • a
    • b
    • c
    • d
    • e

    d

    Explanation: Method overriding requires a method in a child class to use the same method name and same parameter sequence used by one of its ancestors

    Consider the code below. The declared type of s is Staff and it appears as if the adjustSalary(int) operation of the Staff class is invoked.

    void adjustSalary(int byPercent) {
        for (Staff s: staff) {
            s.adjustSalary(byPercent);
        }
    }
    

    However, at runtime s can receive an object of any subclass of Staff. That means the adjustSalary(int) operation of the actual subclass object will be called. If the subclass does not override that operation, the operation defined in the superclass (in this case, Staff class) will be called.

    Static binding (aka early binding): When a method call is resolved at compile time.

    In contrast, overloaded methods are resolved using static binding.

    Paradigms → OOP → Inheritance →

    Overloading

    Method overloading is when there are multiple methods with the same name but different type signatures. Overloading is used to indicate that multiple operations do similar things but take different parameters.

    Type Signature: The type signature of an operation is the type sequence of the parameters. The return type and parameter names are not part of the type signature. However, the parameter order is significant.

    Example:

    Method Type Signature
    int add(int X, int Y) (int, int)
    void add(int A, int B) (int, int)
    void m(int X, double Y) (int, double)
    void m(double X, int Y) (double, int)

    In the case below, the calculate method is overloaded because the two methods have the same name but different type signatures (String) and (int)

    • calculate(String): void
    • calculate(int): void

    Note how the constructor is overloaded in the class below. The method call new Account() is bound to the first constructor at compile time.

    class Account {
    
        Account () {
            // Signature: ()
            ...
        }
    
        Account (String name, String number, double balance) {
            // Signature: (String, String, double)
            ...
        }
    }
    

    Similarly, the calcuateGrade method is overloaded in the code below and a method call calculateGrade("A1213232") is bound to the second implementation, at compile time.

    void calculateGrade (int[] averages) { ... }
    void calculateGrade (String matric) { ... }
    

    W1.3i :

    Paradigms → OOP → Polymorphism → How

    Can explain how substitutability operation overriding, and dynamic binding relates to polymorphism

    Three concepts combine to achieve polymorphism: substitutability, operation overriding, and dynamic binding.

    • Substitutability: Because of substitutability, you can write code that expects object of a parent class and yet use that code with objects of child classes. That is how polymorphism is able to treat objects of different types as one type.
    • Overriding: To get polymorphic behavior from an operation, the operation in the superclass needs to be overridden in each of the subclasses. That is how overriding allows objects of different subclasses to display different behaviors in response to the same method call.
    • Dynamic binding: Calls to overridden methods are bound to the implementation of the actual object's class dynamically during the runtime. That is how the polymorphic code can call the method of the parent class and yet execute the implementation of the child class.

    Which one of these is least related to how OO programs achieve polymorphism?

    (c)

    Explanation: Operation overriding is the one that is related, not operation overloading.

    [W1.4] Java: Collections

    W1.4a :

    C++ to Java → Collections → The Collections Framework

    Can explain the Collections framework

    This section uses extracts from the -- Java Tutorial, with some adaptations.

    A collection — sometimes called a container — is simply an object that groups multiple elements into a single unit. Collections are used to store, retrieve, manipulate, and communicate aggregate data.

    Typically, collections represent data items that form a natural group, such as a poker hand (a collection of cards), a mail folder (a collection of letters), or a telephone directory (a mapping of names to phone numbers).

    The collections framework is a unified architecture for representing and manipulating collections. It contains the following:

    • Interfaces: These are abstract data types that represent collections. Interfaces allow collections to be manipulated independently of the details of their representation.
      Example: the List<E> interface can be used to manipulate list-like collections which may be implemented in different ways such as ArrayList<E> or LinkedList<E>.

    • Implementations: These are the concrete implementations of the collection interfaces. In essence, they are reusable data structures.
      Example: the ArrayList<E> class implements the List<E> interface while the HashMap<K, V> class implements the Map<K, V> interface.

    • Algorithms: These are the methods that perform useful computations, such as searching and sorting, on objects that implement collection interfaces. The algorithms are said to be polymorphic: that is, the same method can be used on many different implementations of the appropriate collection interface.
      Example: the sort(List<E>) method can sort a collection that implements the List<E> interface.

    A well-known example of collections frameworks is the C++ Standard Template Library (STL). Although both are collections frameworks and the syntax look similar, note that there are important philosophical and implementation differences between the two.

    The following list describes the core collection interfaces:

    • Collection — the root of the collection hierarchy. A collection represents a group of objects known as its elements. The Collection interface is the least common denominator that all collections implement and is used to pass collections around and to manipulate them when maximum generality is desired. Some types of collections allow duplicate elements, and others do not. Some are ordered and others are unordered. The Java platform doesn't provide any direct implementations of this interface but provides implementations of more specific subinterfaces, such as Set and List. Also see the Collection API.

    • Set — a collection that cannot contain duplicate elements. This interface models the mathematical set abstraction and is used to represent sets, such as the cards comprising a poker hand, the courses making up a student's schedule, or the processes running on a machine. Also see the Set API.

    • List — an ordered collection (sometimes called a sequence). Lists can contain duplicate elements. The user of a List generally has precise control over where in the list each element is inserted and can access elements by their integer index (position). Also see the List API.

    • Queue — a collection used to hold multiple elements prior to processing. Besides basic Collection operations, a Queue provides additional insertion, extraction, and inspection operations. Also see the Queue API.

    • Map — an object that maps keys to values. A Map cannot contain duplicate keys; each key can map to at most one value. Also see the Map API.

    • Others: Deque, SortedSet, SortedMap

    W1.4b :

    C++ to Java → Collections → The ArrayList Class

    Can use the ArrayList class

    The ArrayList class is a resizable-array implementation of the List interface. Unlike a normal array, an ArrayList can grow in size as you add more items to it. The example below illustrate some of the useful methods of the ArrayList class using an ArrayList of String objects.

    import java.util.ArrayList;
    
    public class ArrayListDemo {
    
        public static void main(String args[]) {
            ArrayList<String> items = new ArrayList<>();
    
            System.out.println("Before adding any items:" + items);
    
            items.add("Apple");
            items.add("Box");
            items.add("Cup");
            items.add("Dart");
            print("After adding four items: " + items);
    
            items.remove("Box"); // remove item "Box"
            print("After removing Box: " + items);
    
            items.add(1, "Banana"); // add "Banana" at index 1
            print("After adding Banana: " + items);
    
            items.add("Egg"); // add "Egg", will be added to the end
            items.add("Cup"); // add another "Cup"
            print("After adding Egg: " + items);
    
            print("Number of items: " + items.size());
    
            print("Index of Cup: " + items.indexOf("Cup"));
            print("Index of Zebra: " + items.indexOf("Zebra"));
    
            print("Item at index 3 is: " + items.get(2));
    
            print("Do we have a Box?: " + items.contains("Box"));
            print("Do we have an Apple?: " + items.contains("Apple"));
    
            items.clear();
            print("After clearing: " + items);
        }
    
        private static void print(String text) {
            System.out.println(text);
        }
    }
    

    Before adding any items:[]
    After adding four items: [Apple, Box, Cup, Dart]
    After removing Box: [Apple, Cup, Dart]
    After adding Banana: [Apple, Banana, Cup, Dart]
    After adding Egg: [Apple, Banana, Cup, Dart, Egg, Cup]
    Number of items: 6
    Index of Cup: 2
    Index of Zebra: -1
    Item at index 3 is: Cup
    Do we have a Box?: false
    Do we have an Apple?: true
    After clearing: []
    

    [Try the above code on Repl.it]

    Add the missing methods to the class given below so that it produces the output given.

    Use an ArrayList to store the numbers.

    public class Main {
    
        //TODO: add your methods here
    
        public static void main(String[] args) {
            System.out.println("Adding numbers to the list");
            addNumber(3);
            addNumber(8);
            addNumber(24);
            System.out.println("The total is: " + getTotal());
            System.out.println("8 in the list : " + isFound(8) );
            System.out.println("5 in the list : " + isFound(5) );
            removeNumber(8);
            System.out.println("The total is: " + getTotal());
        }
    
    }
    

    Adding numbers to the list
    [3]
    [3, 8]
    [3, 8, 24]
    The total is: 35
    8 in the list : true
    5 in the list : false
    [3, 24]
    The total is: 27
    

    Partial solution:

    import java.util.ArrayList;
    
    public class Main {
        private static ArrayList<Integer> numbers = new ArrayList<>();
    
        private static void addNumber(int i) {
            numbers.add(Integer.valueOf(i));
            System.out.println(numbers);
        }
    
        // ...
    
    }
    

    W1.4c :

    C++ to Java → Collections → The HashMap Class

    Can use the HashMap class

    HashMap is an implementation of the Map interface. It allows you to store a collection of key-value pairs. The example below illustrates how to use a HashMap<String, Point> to maintain a list of coordinates and their identifiers e.g., the identifier x1 is used to identify the point 0,0 where x1 is the key and 0,0 is the value.

    import java.awt.Point;
    import java.util.HashMap;
    import java.util.Map;
    
    public class HashMapDemo {
        public static void main(String[] args) {
            HashMap<String, Point> points = new HashMap<>();
    
            // put the key-value pairs in the HashMap
            points.put("x1", new Point(0, 0));
            points.put("x2", new Point(0, 5));
            points.put("x3", new Point(5, 5));
            points.put("x4", new Point(5, 0));
    
            // retrieve a value for a key using the get method
            print("Coordinates of x1: " + pointAsString(points.get("x1")));
    
            // check if a key or a value exists
            print("Key x1 exists? " + points.containsKey("x1"));
            print("Key x1 exists? " + points.containsKey("y1"));
            print("Value (0,0) exists? " + points.containsValue(new Point(0, 0)));
            print("Value (1,2) exists? " + points.containsValue(new Point(1, 2)));
    
            // update the value of a key to a new value
            points.put("x1", new Point(-1,-1));
    
            // iterate over the entries
            for (Map.Entry<String, Point> entry : points.entrySet()) {
                print(entry.getKey() + " = " + pointAsString(entry.getValue()));
            }
    
            print("Number of keys: " + points.size());
            points.clear();
            print("Number of keys after clearing: " + points.size());
    
        }
    
        public static String pointAsString(Point p) {
            return "[" + p.x + "," + p.y + "]";
        }
    
        public static void print(String s) {
            System.out.println(s);
        }
    }
    

    Coordinates of x1: [0,0]
    Key x1 exists? true
    Key x1 exists? false
    Value (0,0) exists? true
    Value (1,2) exists? false
    x1 = [-1,-1]
    x2 = [0,5]
    x3 = [5,5]
    x4 = [5,0]
    Number of keys: 4
    Number of keys after clearing: 0
    

    [Try the above code on Repl.it]

    The class given below keeps track of how many people signup to attend an event on each day of the week. Add the missing methods so that it produces the output given.

    Use an HashMap to store the number of entries for each day.

    public class Main {
        private static HashMap<String, Integer> roster = new HashMap<>();
    
        //TODO: add your methods here
    
        public static void main(String[] args) {
            addToRoster("Monday"); // i.e., one person signed up for Monday
            addToRoster("Wednesday"); // i.e., one person signed up for Wednesday
            addToRoster("Wednesday"); // i.e., another person signed up for Wednesday
            addToRoster("Friday");
            addToRoster("Monday");
            printRoster();
        }
    
    }
    

    Monday => 2
    Friday => 1
    Wednesday => 2
    

    Partial solution:

    import java.util.HashMap;
    import java.util.Map;
    
    public class Main {
        private static HashMap<String, Integer> roster = new HashMap<>();
    
        private static void addToRoster(String day) {
            if (roster.containsKey(day)){
                Integer newValue = Integer.valueOf(roster.get(day).intValue() + 1);
                roster.put(day, newValue);
            } else {
                roster.put(day, Integer.valueOf(1));
            }
        }
    
        // ...
    }
    

    [W1.5] Exception Handling

    W1.5a :

    Implementation → Error Handling → Introduction → What

    Can explain error handling

    Well-written applications include error-handling code that allows them to recover gracefully from unexpected errors. When an error occurs, the application may need to request user intervention, or it may be able to recover on its own. In extreme cases, the application may log the user off or shut down the system. --Microsoft

    W1.5b :

    Implementation → Error Handling → Exceptions → What

    Can explain exceptions

    Exceptions are used to deal with 'unusual' but not entirely unexpected situations that the program might encounter at run time.

    Exception:

    The term exception is shorthand for the phrase "exceptional event." An exception is an event, which occurs during the execution of a program, that disrupts the normal flow of the program's instructions. –- Java Tutorial (Oracle Inc.)

    Examples:

    • A network connection encounters a timeout due to a slow server.
    • The code tries to read a file from the hard disk but the file is corrupted and cannot be read.

    W1.5c :

    C++ to Java → Exceptions → What are Exceptions?

    Can explain Java Exceptions

    Given below is an extract from the -- Java Tutorial, with some adaptations.

    There are three basic categories of exceptions In Java:

    • Checked exceptions: exceptional conditions that a well-written application should anticipate and recover from. All exceptions are checked exceptions, except for Error, RuntimeException, and their subclasses.

    Suppose an application prompts a user for an input file name, then opens the file by passing the name to the constructor for java.io.FileReader. Normally, the user provides the name of an existing, readable file, so the construction of the FileReader object succeeds, and the execution of the application proceeds normally. But sometimes the user supplies the name of a nonexistent file, and the constructor throws java.io.FileNotFoundException. A well-written program will catch this exception and notify the user of the mistake, possibly prompting for a corrected file name.

    • Errors: exceptional conditions that are external to the application, and that the application usually cannot anticipate or recover from. Errors are those exceptions indicated by Error and its subclasses.

    Suppose that an application successfully opens a file for input, but is unable to read the file because of a hardware or system malfunction. The unsuccessful read will throw java.io.IOError. An application might choose to catch this exception, in order to notify the user of the problem — but it also might make sense for the program to print a stack trace and exit.

    • Runtime exceptions: conditions that are internal to the application, and that the application usually cannot anticipate or recover from. Runtime exceptions are those indicated by RuntimeException and its subclasses. These usually indicate programming bugs, such as logic errors or improper use of an API.

    Consider the application described previously that passes a file name to the constructor for FileReader. If a logic error causes a null to be passed to the constructor, the constructor will throw NullPointerException. The application can catch this exception, but it probably makes more sense to eliminate the bug that caused the exception to occur.

    Errors and runtime exceptions are collectively known as unchecked exceptions.

    W1.5d :

    Implementation → Error Handling → Exceptions → How

    Can explain how exception handling is done typically

    Most languages allow code that encountered an "exceptional" situation to encapsulate details of the situation in an Exception object and throw/raise that object so that another piece of code can catch it and deal with it. This is especially useful when the code that encountered the unusual situation does not know how to deal with it.

    The extract below from the -- Java Tutorial (with slight adaptations) explains how exceptions are typically handled.

    When an error occurs at some point in the execution, the code being executed creates an exception object and hands it off to the runtime system. The exception object contains information about the error, including its type and the state of the program when the error occurred. Creating an exception object and handing it to the runtime system is called throwing an exception.

    After a method throws an exception, the runtime system attempts to find something to handle it in the call stack. The runtime system searches the call stack for a method that contains a block of code that can handle the exception. This block of code is called an exception handler. The search begins with the method in which the error occurred and proceeds through the call stack in the reverse order in which the methods were called. When an appropriate handler is found, the runtime system passes the exception to the handler. An exception handler is considered appropriate if the type of the exception object thrown matches the type that can be handled by the handler.

    The exception handler chosen is said to catch the exception. If the runtime system exhaustively searches all the methods on the call stack without finding an appropriate exception handler, the program terminates.

    Advantages of exception handling in this way:

    • The ability to propagate error information through the call stack.
    • The separation of code that deals with 'unusual' situations from the code that does the 'usual' work.

    Which are benefits of exceptions?

    • a. Exceptions allow us to separate normal code from error handling code.
    • b. Exceptions can prevent problems that happen in the environment.
    • c. Exceptions allow us to handle in one location an error raised in another location.

    (a) (c)

    Explanation: Exceptions cannot prevent problems in the environment. They can only be used to handle and recover from such problems.

    W1.5e :

    C++ to Java → Exceptions → How to Use Exceptions

    Can use Java Exceptions

    The content below uses extracts from the -- Java Tutorial, with some adaptations.

    A program can catch exceptions by using a combination of the try, catch blocks.

    • The try block identifies a block of code in which an exception can occur.
    • The catch block identifies a block of code, known as an exception handler, that can handle a particular type of exception.

    The writeList() method below calls a method process() that can cause two type of exceptions. It uses a try-catch construct to deal with each exception.

    public void writeList() {
        print("starting method");
        try {
            print("starting process");
            process();
            print("finishing process");
    
        } catch (IndexOutOfBoundsException e) {
            print("caught IOOBE");
    
        } catch (IOException e) {
            print("caught IOE");
    
        }
        print("finishing method");
    }
    

    Some possible outputs:

    No exceptions IOException IndexOutOfBoundsException
    starting method
    starting process
    finishing process
    finishing method
    starting method
    starting process
    finishing process
    caught IOE
    finishing method
    starting method
    starting process
    finishing process
    caught IOOBE
    finishing method

    You can use a finally block to specify code that is guaranteed to execute with or without the exception. This is the right place to close files, recover resources, and otherwise clean up after the code enclosed in the try block.

    The writeList() method below has a finally block:

    public void writeList() {
        print("starting method");
        try {
            print("starting process");
            process();
            print("finishing process");
    
        } catch (IndexOutOfBoundsException e) {
            print("caught IOOBE");
    
        } catch (IOException e) {
            print("caught IOE");
    
        } finally {
            // clean up
            print("cleaning up");
        }
        print("finishing method");
    }
    

    Some possible outputs:

    No exceptions IOException IndexOutOfBoundsException
    starting method
    starting process
    finishing process
    cleaning up
    finishing method
    starting method
    starting process
    finishing process
    caught IOE
    cleaning up
    finishing method
    starting method
    starting process
    finishing process
    caught IOOBE
    cleaning up
    finishing method
    • The try statement should contain at least one catch block or a finally block and may have multiple catch blocks.

    • The class of the exception object indicates the type of exception thrown. The exception object can contain further information about the error, including an error message.

    You can use the throw statement to throw an exception. The throw statement requires a throwable object as the argument.

    Here's an example of a throw statement.

    if (size == 0) {
        throw new EmptyStackException();
    }
    

    In Java, Checked exceptions are subject to the Catch or Specify Requirement: code that might throw checked exceptions must be enclosed by either of the following:

    • A try statement that catches the exception. The try must provide a handler for the exception.
    • A method that specifies that it can throw the exception. The method must provide a throws clause that lists the exception.

    Unchecked exceptions are not required to follow to the Catch or Specify Requirement but you can apply the requirement to them too.

    Here's an example of a method specifying that it throws certain checked exceptions:

    public void writeList() throws IOException, IndexOutOfBoundsException {
        print("starting method");
        process();
        print("finishing method");
    }
    
    Some possible outputs:
    
    No exceptions IOException IndexOutOfBoundsException
    starting method
    finishing method
    starting method
    finishing method
    starting method
    finishing method

    Java comes with a collection of built-in exception classes that you can use. When they are not enough, it is possible to create your own exception classes.

    The Main class below parses a string descriptor of a rectangle of the format "WIDTHxHEIGHT" e.g., "3x4" and prints the area of the rectangle.

    public class Main {
    
        public static void printArea(String descriptor){
            //TODO: modify the code below
            System.out.println(descriptor + "=" + calculateArea(descriptor));
        }
    
        private static int calculateArea(String descriptor) {
            //TODO: modify the code below
            String[] dimensions = descriptor.split("x");
            return Integer.parseInt(dimensions[0]) * Integer.parseInt(dimensions[1]);
        }
    
        public static void main(String[] args) {
            printArea("3x4");
            printArea("5x5");
        }
    }
    

    3x4=12
    5x5=25
    
    1. Update the code of printArea to print an error message if WIDTH and/or HEIGHT are not numbers e.g., "Ax4"
      calculateArea will throw the unchecked exception NumberFormatException if the code tries to parse a non-number to an integer.

    2. Update the code of printArea to print an error message if the descriptor is missing WIDTH and/or HEIGHT e.g., "x4"
      calculateArea will throw the unchecked exception IndexOutOfBoundsException if one or both dimensions are missing.

    3. Update the code of calculateArea to throw the checked exception IllegalShapeException if there are more than 2 dimensions e.g., "5x4x3" and update the printArea to print an error message for those cases. Here is the code for the IllegalShapeException.java

    public class IllegalShapeException extends Exception {
      //no other code needed
    }
    

    Here is the expected behavior after you have done the above changes:

    public class Main {
    
        //...
    
        public static void main(String[] args) {
            printArea("3x4");
            printArea("3xy");
            printArea("3x");
            printArea("3");
            printArea("3x4x5");
        }
    }
    

    3x4=12
    WIDTH or HEIGHT is not a number: 3xy
    WIDTH or HEIGHT is missing: 3x
    WIDTH or HEIGHT is missing: 3
    Too many dimensions: 3x4x5
    
    public class Main {
    
        public static void printArea(String descriptor){
            try {
                System.out.println(descriptor + "=" + calculateArea(descriptor));
            } catch (NumberFormatException e) {
                System.out.println("WIDTH or HEIGHT is not a number: " + descriptor);
            } // add more catch blocks here
        }
    
        private static int calculateArea(String descriptor) throws IllegalShapeException {
            String[] dimensions = descriptor.split("x");
    
            //throw IllegalShapeException here if dimensions.length > 2
    
            return Integer.parseInt(dimensions[0]) * Integer.parseInt(dimensions[1]);
        }
    
    
    }
    

    W1.5f :

    Implementation → Error Handling → Exceptions → When

    Can avoid using exceptions to control normal workflow

    In general, use exceptions only for 'unusual' conditions. Use normal return statements to pass control to the caller for conditions that are 'normal'.